IM 10Q

A 74-year-old man presents to your office for a routine. He has no present complaint. His medical history is significant for right knee osteoarthritis. He takes naproxen occasionally, to relieve knee pain. He does not smoke or consume alcohol. His BP is 165/75 mmHg and PR is 70/min. The physical examination showed a mild systolic ejection type murmur at the base of the heat to the right. An E-KG revealed left ventricular hypertrophy and secondary ST segment and T wave change. Moderate left ventricular hypertrophy, without any flow abnormalities, was demonstrated on echocardiography. The ejection fraction was 60%. What is the most probable cause of hypertension in this patient?
Rigidity of the arterial wall
Elevated plasma renin activity
Aortic insufficiency
Increased cardiac output
Increased intravascular volume
 
 
{"name":"IM 10Q", "url":"https://www.quiz-maker.com/QPREVIEW","txt":"A 74-year-old man presents to your office for a routine. He has no present complaint. His medical history is significant for right knee osteoarthritis. He takes naproxen occasionally, to relieve knee pain. He does not smoke or consume alcohol. His BP is 165\/75 mmHg and PR is 70\/min. The physical examination showed a mild systolic ejection type murmur at the base of the heat to the right. An E-KG revealed left ventricular hypertrophy and secondary ST segment and T wave change. Moderate left ventricular hypertrophy, without any flow abnormalities, was demonstrated on echocardiography. The ejection fraction was 60%. What is the most probable cause of hypertension in this patient?","img":"https://www.quiz-maker.com/3012/images/ogquiz.png"}
Powered by: Quiz Maker